Proving sum of recurrence sequence converges to $2^-55$

The name of the pictureThe name of the pictureThe name of the pictureClash Royale CLAN TAG#URR8PPP











up vote
0
down vote

favorite
1












$$24a(n)=26a(n-1)-9a(n-2)+a(n-3)$$
$$a(0)=46, a(1)=8, a(2)=1$$
$$sumlimits_k=3^inftya(k)=2^-55$$
How can I prove it?










share|cite|improve this question

























    up vote
    0
    down vote

    favorite
    1












    $$24a(n)=26a(n-1)-9a(n-2)+a(n-3)$$
    $$a(0)=46, a(1)=8, a(2)=1$$
    $$sumlimits_k=3^inftya(k)=2^-55$$
    How can I prove it?










    share|cite|improve this question























      up vote
      0
      down vote

      favorite
      1









      up vote
      0
      down vote

      favorite
      1






      1





      $$24a(n)=26a(n-1)-9a(n-2)+a(n-3)$$
      $$a(0)=46, a(1)=8, a(2)=1$$
      $$sumlimits_k=3^inftya(k)=2^-55$$
      How can I prove it?










      share|cite|improve this question













      $$24a(n)=26a(n-1)-9a(n-2)+a(n-3)$$
      $$a(0)=46, a(1)=8, a(2)=1$$
      $$sumlimits_k=3^inftya(k)=2^-55$$
      How can I prove it?







      summation proof-writing recurrence-relations






      share|cite|improve this question













      share|cite|improve this question











      share|cite|improve this question




      share|cite|improve this question










      asked Sep 4 at 11:11









      user565184

      285




      285




















          3 Answers
          3






          active

          oldest

          votes

















          up vote
          2
          down vote



          accepted










          $$24r^3=26r^2-9r+1$$



          solutions
          $$r=frac12,frac13,frac14$$



          $$a_n=xleft(frac12right)^n+yleft(frac13right)^n+zleft(frac14right)^n$$
          Determine $x,y,z$ using base conditions



          $$a_0=46=x+y+z$$
          $$a_1=8=x/2+y/3+z/4$$
          $$a_2=1=x/4+y/9+z/16$$



          $$fbox
          x=4, y=-54,z=96
          $$



          $$a_n=4left(frac12right)^n-54left(frac13right)^n+96left(frac14right)^n$$



          $$sumlimits_k=3^inftya(k)=sumlimits_k=3^inftyleft(4left(frac12right)^k-54left(frac13right)^k+96left(frac14right)^kright)$$



          $$sum_k=3^infty4left(frac12right)^k=4sum_k=3^inftyleft(frac12right)^k=4cdotfracleft(frac12right)^31-frac12=1$$



          $$54sum_k=3^inftyleft(frac13right)^k=54fracleft(frac13right)^31-frac13=3$$
          $$96sum_k=3^inftyleft(frac14right)^k=96cdotfracleft(frac14right)^31-frac14=2$$



          $$sumlimits_k=3^inftya_k=1-3+2=0$$






          share|cite|improve this answer






















          • Do you really get $2^-55$ in the end? WA disagrees.
            – lhf
            Sep 4 at 11:48











          • @lhf, you are right, it looks like solution is $2^-47$.
            – user565184
            Sep 4 at 12:05










          • @user565184, are you summing this with a computer program? This can be misleading.
            – lhf
            Sep 4 at 12:06











          • @lhf, by hand, of course. Thank you for answer! Sorry for my bad sense of humor.
            – user565184
            Sep 4 at 12:08










          • Answer is not $2^-55$ for sure
            – Deepesh Meena
            Sep 4 at 12:10

















          up vote
          2
          down vote













          Let $S=sumlimits_k=3^inftya(k)$. Then
          $$
          24S = 26(S+a(2))-9(S+a(1)+a(2))+(S+a(0)+a(1)+a(2))
          $$
          This gives $S=0$.






          share|cite|improve this answer



























            up vote
            1
            down vote













            Consider the generating function $$f(x)=a_0x^0+a_1x^1+a_2x^2+cdots+a_nx^n+cdots.tag1$$



            We have $$f(x)cdot x^3=a_0x^3+a_1x^4+a_2x^5+cdots+a_nx^n+3+cdots;tag2$$
            $$f(x)cdot x^2=a_0x^2+a_1x^3+a_2x^4+cdots+a_nx^n+2+cdots;tag3$$
            $$f(x)cdot x=a_0x^1+a_1x^2+a_2x^3+cdots+a_nx^n+1+cdots.tag4$$
            Thus, by $(2)-9times(3)+26times(4)-(1)$, we obtain
            beginalign*
            f(x)(x^3-9x^2+26x-1)&=-9a_0x^2+26a_0x^1+26a_1x^2-a_0x^0-a_1x^1-a_2x^2\
            &=-207x^2+1188x-46
            endalign*



            Hence $$boxedf(x)=frac-207x^2+1188x-46x^3-9x^2+26x-1.$$



            Thus $$sum_k=0^infty a_k=f(1)=55.$$



            As a result, $$sum_k=3^infty a_k=sum_k=0^infty a_k-sum_k=0^2 a_k=55-55=0.$$






            share|cite|improve this answer






















            • Very nice point of view, thank you for answer!
              – user565184
              Sep 4 at 15:41










            Your Answer




            StackExchange.ifUsing("editor", function ()
            return StackExchange.using("mathjaxEditing", function ()
            StackExchange.MarkdownEditor.creationCallbacks.add(function (editor, postfix)
            StackExchange.mathjaxEditing.prepareWmdForMathJax(editor, postfix, [["$", "$"], ["\\(","\\)"]]);
            );
            );
            , "mathjax-editing");

            StackExchange.ready(function()
            var channelOptions =
            tags: "".split(" "),
            id: "69"
            ;
            initTagRenderer("".split(" "), "".split(" "), channelOptions);

            StackExchange.using("externalEditor", function()
            // Have to fire editor after snippets, if snippets enabled
            if (StackExchange.settings.snippets.snippetsEnabled)
            StackExchange.using("snippets", function()
            createEditor();
            );

            else
            createEditor();

            );

            function createEditor()
            StackExchange.prepareEditor(
            heartbeatType: 'answer',
            convertImagesToLinks: true,
            noModals: false,
            showLowRepImageUploadWarning: true,
            reputationToPostImages: 10,
            bindNavPrevention: true,
            postfix: "",
            noCode: true, onDemand: true,
            discardSelector: ".discard-answer"
            ,immediatelyShowMarkdownHelp:true
            );



            );













             

            draft saved


            draft discarded


















            StackExchange.ready(
            function ()
            StackExchange.openid.initPostLogin('.new-post-login', 'https%3a%2f%2fmath.stackexchange.com%2fquestions%2f2904912%2fproving-sum-of-recurrence-sequence-converges-to-2-55%23new-answer', 'question_page');

            );

            Post as a guest






























            3 Answers
            3






            active

            oldest

            votes








            3 Answers
            3






            active

            oldest

            votes









            active

            oldest

            votes






            active

            oldest

            votes








            up vote
            2
            down vote



            accepted










            $$24r^3=26r^2-9r+1$$



            solutions
            $$r=frac12,frac13,frac14$$



            $$a_n=xleft(frac12right)^n+yleft(frac13right)^n+zleft(frac14right)^n$$
            Determine $x,y,z$ using base conditions



            $$a_0=46=x+y+z$$
            $$a_1=8=x/2+y/3+z/4$$
            $$a_2=1=x/4+y/9+z/16$$



            $$fbox
            x=4, y=-54,z=96
            $$



            $$a_n=4left(frac12right)^n-54left(frac13right)^n+96left(frac14right)^n$$



            $$sumlimits_k=3^inftya(k)=sumlimits_k=3^inftyleft(4left(frac12right)^k-54left(frac13right)^k+96left(frac14right)^kright)$$



            $$sum_k=3^infty4left(frac12right)^k=4sum_k=3^inftyleft(frac12right)^k=4cdotfracleft(frac12right)^31-frac12=1$$



            $$54sum_k=3^inftyleft(frac13right)^k=54fracleft(frac13right)^31-frac13=3$$
            $$96sum_k=3^inftyleft(frac14right)^k=96cdotfracleft(frac14right)^31-frac14=2$$



            $$sumlimits_k=3^inftya_k=1-3+2=0$$






            share|cite|improve this answer






















            • Do you really get $2^-55$ in the end? WA disagrees.
              – lhf
              Sep 4 at 11:48











            • @lhf, you are right, it looks like solution is $2^-47$.
              – user565184
              Sep 4 at 12:05










            • @user565184, are you summing this with a computer program? This can be misleading.
              – lhf
              Sep 4 at 12:06











            • @lhf, by hand, of course. Thank you for answer! Sorry for my bad sense of humor.
              – user565184
              Sep 4 at 12:08










            • Answer is not $2^-55$ for sure
              – Deepesh Meena
              Sep 4 at 12:10














            up vote
            2
            down vote



            accepted










            $$24r^3=26r^2-9r+1$$



            solutions
            $$r=frac12,frac13,frac14$$



            $$a_n=xleft(frac12right)^n+yleft(frac13right)^n+zleft(frac14right)^n$$
            Determine $x,y,z$ using base conditions



            $$a_0=46=x+y+z$$
            $$a_1=8=x/2+y/3+z/4$$
            $$a_2=1=x/4+y/9+z/16$$



            $$fbox
            x=4, y=-54,z=96
            $$



            $$a_n=4left(frac12right)^n-54left(frac13right)^n+96left(frac14right)^n$$



            $$sumlimits_k=3^inftya(k)=sumlimits_k=3^inftyleft(4left(frac12right)^k-54left(frac13right)^k+96left(frac14right)^kright)$$



            $$sum_k=3^infty4left(frac12right)^k=4sum_k=3^inftyleft(frac12right)^k=4cdotfracleft(frac12right)^31-frac12=1$$



            $$54sum_k=3^inftyleft(frac13right)^k=54fracleft(frac13right)^31-frac13=3$$
            $$96sum_k=3^inftyleft(frac14right)^k=96cdotfracleft(frac14right)^31-frac14=2$$



            $$sumlimits_k=3^inftya_k=1-3+2=0$$






            share|cite|improve this answer






















            • Do you really get $2^-55$ in the end? WA disagrees.
              – lhf
              Sep 4 at 11:48











            • @lhf, you are right, it looks like solution is $2^-47$.
              – user565184
              Sep 4 at 12:05










            • @user565184, are you summing this with a computer program? This can be misleading.
              – lhf
              Sep 4 at 12:06











            • @lhf, by hand, of course. Thank you for answer! Sorry for my bad sense of humor.
              – user565184
              Sep 4 at 12:08










            • Answer is not $2^-55$ for sure
              – Deepesh Meena
              Sep 4 at 12:10












            up vote
            2
            down vote



            accepted







            up vote
            2
            down vote



            accepted






            $$24r^3=26r^2-9r+1$$



            solutions
            $$r=frac12,frac13,frac14$$



            $$a_n=xleft(frac12right)^n+yleft(frac13right)^n+zleft(frac14right)^n$$
            Determine $x,y,z$ using base conditions



            $$a_0=46=x+y+z$$
            $$a_1=8=x/2+y/3+z/4$$
            $$a_2=1=x/4+y/9+z/16$$



            $$fbox
            x=4, y=-54,z=96
            $$



            $$a_n=4left(frac12right)^n-54left(frac13right)^n+96left(frac14right)^n$$



            $$sumlimits_k=3^inftya(k)=sumlimits_k=3^inftyleft(4left(frac12right)^k-54left(frac13right)^k+96left(frac14right)^kright)$$



            $$sum_k=3^infty4left(frac12right)^k=4sum_k=3^inftyleft(frac12right)^k=4cdotfracleft(frac12right)^31-frac12=1$$



            $$54sum_k=3^inftyleft(frac13right)^k=54fracleft(frac13right)^31-frac13=3$$
            $$96sum_k=3^inftyleft(frac14right)^k=96cdotfracleft(frac14right)^31-frac14=2$$



            $$sumlimits_k=3^inftya_k=1-3+2=0$$






            share|cite|improve this answer














            $$24r^3=26r^2-9r+1$$



            solutions
            $$r=frac12,frac13,frac14$$



            $$a_n=xleft(frac12right)^n+yleft(frac13right)^n+zleft(frac14right)^n$$
            Determine $x,y,z$ using base conditions



            $$a_0=46=x+y+z$$
            $$a_1=8=x/2+y/3+z/4$$
            $$a_2=1=x/4+y/9+z/16$$



            $$fbox
            x=4, y=-54,z=96
            $$



            $$a_n=4left(frac12right)^n-54left(frac13right)^n+96left(frac14right)^n$$



            $$sumlimits_k=3^inftya(k)=sumlimits_k=3^inftyleft(4left(frac12right)^k-54left(frac13right)^k+96left(frac14right)^kright)$$



            $$sum_k=3^infty4left(frac12right)^k=4sum_k=3^inftyleft(frac12right)^k=4cdotfracleft(frac12right)^31-frac12=1$$



            $$54sum_k=3^inftyleft(frac13right)^k=54fracleft(frac13right)^31-frac13=3$$
            $$96sum_k=3^inftyleft(frac14right)^k=96cdotfracleft(frac14right)^31-frac14=2$$



            $$sumlimits_k=3^inftya_k=1-3+2=0$$







            share|cite|improve this answer














            share|cite|improve this answer



            share|cite|improve this answer








            edited Sep 4 at 12:17

























            answered Sep 4 at 11:19









            Deepesh Meena

            3,8012825




            3,8012825











            • Do you really get $2^-55$ in the end? WA disagrees.
              – lhf
              Sep 4 at 11:48











            • @lhf, you are right, it looks like solution is $2^-47$.
              – user565184
              Sep 4 at 12:05










            • @user565184, are you summing this with a computer program? This can be misleading.
              – lhf
              Sep 4 at 12:06











            • @lhf, by hand, of course. Thank you for answer! Sorry for my bad sense of humor.
              – user565184
              Sep 4 at 12:08










            • Answer is not $2^-55$ for sure
              – Deepesh Meena
              Sep 4 at 12:10
















            • Do you really get $2^-55$ in the end? WA disagrees.
              – lhf
              Sep 4 at 11:48











            • @lhf, you are right, it looks like solution is $2^-47$.
              – user565184
              Sep 4 at 12:05










            • @user565184, are you summing this with a computer program? This can be misleading.
              – lhf
              Sep 4 at 12:06











            • @lhf, by hand, of course. Thank you for answer! Sorry for my bad sense of humor.
              – user565184
              Sep 4 at 12:08










            • Answer is not $2^-55$ for sure
              – Deepesh Meena
              Sep 4 at 12:10















            Do you really get $2^-55$ in the end? WA disagrees.
            – lhf
            Sep 4 at 11:48





            Do you really get $2^-55$ in the end? WA disagrees.
            – lhf
            Sep 4 at 11:48













            @lhf, you are right, it looks like solution is $2^-47$.
            – user565184
            Sep 4 at 12:05




            @lhf, you are right, it looks like solution is $2^-47$.
            – user565184
            Sep 4 at 12:05












            @user565184, are you summing this with a computer program? This can be misleading.
            – lhf
            Sep 4 at 12:06





            @user565184, are you summing this with a computer program? This can be misleading.
            – lhf
            Sep 4 at 12:06













            @lhf, by hand, of course. Thank you for answer! Sorry for my bad sense of humor.
            – user565184
            Sep 4 at 12:08




            @lhf, by hand, of course. Thank you for answer! Sorry for my bad sense of humor.
            – user565184
            Sep 4 at 12:08












            Answer is not $2^-55$ for sure
            – Deepesh Meena
            Sep 4 at 12:10




            Answer is not $2^-55$ for sure
            – Deepesh Meena
            Sep 4 at 12:10










            up vote
            2
            down vote













            Let $S=sumlimits_k=3^inftya(k)$. Then
            $$
            24S = 26(S+a(2))-9(S+a(1)+a(2))+(S+a(0)+a(1)+a(2))
            $$
            This gives $S=0$.






            share|cite|improve this answer
























              up vote
              2
              down vote













              Let $S=sumlimits_k=3^inftya(k)$. Then
              $$
              24S = 26(S+a(2))-9(S+a(1)+a(2))+(S+a(0)+a(1)+a(2))
              $$
              This gives $S=0$.






              share|cite|improve this answer






















                up vote
                2
                down vote










                up vote
                2
                down vote









                Let $S=sumlimits_k=3^inftya(k)$. Then
                $$
                24S = 26(S+a(2))-9(S+a(1)+a(2))+(S+a(0)+a(1)+a(2))
                $$
                This gives $S=0$.






                share|cite|improve this answer












                Let $S=sumlimits_k=3^inftya(k)$. Then
                $$
                24S = 26(S+a(2))-9(S+a(1)+a(2))+(S+a(0)+a(1)+a(2))
                $$
                This gives $S=0$.







                share|cite|improve this answer












                share|cite|improve this answer



                share|cite|improve this answer










                answered Sep 4 at 11:50









                lhf

                157k9161372




                157k9161372




















                    up vote
                    1
                    down vote













                    Consider the generating function $$f(x)=a_0x^0+a_1x^1+a_2x^2+cdots+a_nx^n+cdots.tag1$$



                    We have $$f(x)cdot x^3=a_0x^3+a_1x^4+a_2x^5+cdots+a_nx^n+3+cdots;tag2$$
                    $$f(x)cdot x^2=a_0x^2+a_1x^3+a_2x^4+cdots+a_nx^n+2+cdots;tag3$$
                    $$f(x)cdot x=a_0x^1+a_1x^2+a_2x^3+cdots+a_nx^n+1+cdots.tag4$$
                    Thus, by $(2)-9times(3)+26times(4)-(1)$, we obtain
                    beginalign*
                    f(x)(x^3-9x^2+26x-1)&=-9a_0x^2+26a_0x^1+26a_1x^2-a_0x^0-a_1x^1-a_2x^2\
                    &=-207x^2+1188x-46
                    endalign*



                    Hence $$boxedf(x)=frac-207x^2+1188x-46x^3-9x^2+26x-1.$$



                    Thus $$sum_k=0^infty a_k=f(1)=55.$$



                    As a result, $$sum_k=3^infty a_k=sum_k=0^infty a_k-sum_k=0^2 a_k=55-55=0.$$






                    share|cite|improve this answer






















                    • Very nice point of view, thank you for answer!
                      – user565184
                      Sep 4 at 15:41














                    up vote
                    1
                    down vote













                    Consider the generating function $$f(x)=a_0x^0+a_1x^1+a_2x^2+cdots+a_nx^n+cdots.tag1$$



                    We have $$f(x)cdot x^3=a_0x^3+a_1x^4+a_2x^5+cdots+a_nx^n+3+cdots;tag2$$
                    $$f(x)cdot x^2=a_0x^2+a_1x^3+a_2x^4+cdots+a_nx^n+2+cdots;tag3$$
                    $$f(x)cdot x=a_0x^1+a_1x^2+a_2x^3+cdots+a_nx^n+1+cdots.tag4$$
                    Thus, by $(2)-9times(3)+26times(4)-(1)$, we obtain
                    beginalign*
                    f(x)(x^3-9x^2+26x-1)&=-9a_0x^2+26a_0x^1+26a_1x^2-a_0x^0-a_1x^1-a_2x^2\
                    &=-207x^2+1188x-46
                    endalign*



                    Hence $$boxedf(x)=frac-207x^2+1188x-46x^3-9x^2+26x-1.$$



                    Thus $$sum_k=0^infty a_k=f(1)=55.$$



                    As a result, $$sum_k=3^infty a_k=sum_k=0^infty a_k-sum_k=0^2 a_k=55-55=0.$$






                    share|cite|improve this answer






















                    • Very nice point of view, thank you for answer!
                      – user565184
                      Sep 4 at 15:41












                    up vote
                    1
                    down vote










                    up vote
                    1
                    down vote









                    Consider the generating function $$f(x)=a_0x^0+a_1x^1+a_2x^2+cdots+a_nx^n+cdots.tag1$$



                    We have $$f(x)cdot x^3=a_0x^3+a_1x^4+a_2x^5+cdots+a_nx^n+3+cdots;tag2$$
                    $$f(x)cdot x^2=a_0x^2+a_1x^3+a_2x^4+cdots+a_nx^n+2+cdots;tag3$$
                    $$f(x)cdot x=a_0x^1+a_1x^2+a_2x^3+cdots+a_nx^n+1+cdots.tag4$$
                    Thus, by $(2)-9times(3)+26times(4)-(1)$, we obtain
                    beginalign*
                    f(x)(x^3-9x^2+26x-1)&=-9a_0x^2+26a_0x^1+26a_1x^2-a_0x^0-a_1x^1-a_2x^2\
                    &=-207x^2+1188x-46
                    endalign*



                    Hence $$boxedf(x)=frac-207x^2+1188x-46x^3-9x^2+26x-1.$$



                    Thus $$sum_k=0^infty a_k=f(1)=55.$$



                    As a result, $$sum_k=3^infty a_k=sum_k=0^infty a_k-sum_k=0^2 a_k=55-55=0.$$






                    share|cite|improve this answer














                    Consider the generating function $$f(x)=a_0x^0+a_1x^1+a_2x^2+cdots+a_nx^n+cdots.tag1$$



                    We have $$f(x)cdot x^3=a_0x^3+a_1x^4+a_2x^5+cdots+a_nx^n+3+cdots;tag2$$
                    $$f(x)cdot x^2=a_0x^2+a_1x^3+a_2x^4+cdots+a_nx^n+2+cdots;tag3$$
                    $$f(x)cdot x=a_0x^1+a_1x^2+a_2x^3+cdots+a_nx^n+1+cdots.tag4$$
                    Thus, by $(2)-9times(3)+26times(4)-(1)$, we obtain
                    beginalign*
                    f(x)(x^3-9x^2+26x-1)&=-9a_0x^2+26a_0x^1+26a_1x^2-a_0x^0-a_1x^1-a_2x^2\
                    &=-207x^2+1188x-46
                    endalign*



                    Hence $$boxedf(x)=frac-207x^2+1188x-46x^3-9x^2+26x-1.$$



                    Thus $$sum_k=0^infty a_k=f(1)=55.$$



                    As a result, $$sum_k=3^infty a_k=sum_k=0^infty a_k-sum_k=0^2 a_k=55-55=0.$$







                    share|cite|improve this answer














                    share|cite|improve this answer



                    share|cite|improve this answer








                    edited Sep 4 at 13:58

























                    answered Sep 4 at 13:52









                    mengdie1982

                    3,824216




                    3,824216











                    • Very nice point of view, thank you for answer!
                      – user565184
                      Sep 4 at 15:41
















                    • Very nice point of view, thank you for answer!
                      – user565184
                      Sep 4 at 15:41















                    Very nice point of view, thank you for answer!
                    – user565184
                    Sep 4 at 15:41




                    Very nice point of view, thank you for answer!
                    – user565184
                    Sep 4 at 15:41

















                     

                    draft saved


                    draft discarded















































                     


                    draft saved


                    draft discarded














                    StackExchange.ready(
                    function ()
                    StackExchange.openid.initPostLogin('.new-post-login', 'https%3a%2f%2fmath.stackexchange.com%2fquestions%2f2904912%2fproving-sum-of-recurrence-sequence-converges-to-2-55%23new-answer', 'question_page');

                    );

                    Post as a guest













































































                    這個網誌中的熱門文章

                    Is there any way to eliminate the singular point to solve this integral by hand or by approximations?

                    Why am i infinitely getting the same tweet with the Twitter Search API?

                    Carbon dioxide